(Quantum Mechanics) Prove that <p> = m (d<x>/dt)

Take a look at this topicIn summary, the conversation discusses using the Schrödinger equation and the complex conjugate to derive the expectation position in quantum mechanics. The process involves taking the time derivative of x\Psi\Psi^* and using integration by parts. It is assumed that the wave function and its derivative approach 0 as x goes to ±∞ for the equation to work. The conversation also addresses the importance of having the wave function vanish at infinity in order for it to be normalizable.
  • #1
emol1414
18
0

Homework Statement


Prove that [itex] <p> = m \frac{d<x>}{dt} [/itex]

Homework Equations


Schrödinger Equation: [itex]i\hbar[/itex] [itex]\frac{\partial \Psi} {\partial x}[/itex] = -[itex] \frac{\hbar^2}{2m}[/itex] [itex]\frac{\partial^2 \Psi}{\partial x^2}[/itex] + [itex]V{} \Psi[/itex]

Respective complex conjugate from equation above

Expectation Position: <x> = [itex]\int_{-\infty}^{+\infty} x\Psi {\Psi}^*[/itex] dx

The Attempt at a Solution


Derive <x> with respect to t... with V real, we know that V = V*, and after some basic steps we get:

[itex]\frac {d<x>}{dt}[/itex] = [itex]\frac{i \hbar}{2m}[/itex] [itex]\int[/itex] [itex]dx[/itex] [itex]x[/itex][[itex]\Psi^*[/itex][itex](\frac{\partial^2 \Psi}{\partial x^2}[/itex]) - [itex]\Psi[/itex] [itex](\frac{\partial^2 \Psi^*}{\partial x^2})[/itex]]

Then my problem is with the integration by parts... for
[itex] \int_{a}^{b}[/itex] [itex] f \frac{dg}{dx} dx [/itex] = [itex] fg [/itex] [itex]{|}^{b}_{a}[/itex] - [itex] \int_{a}^{b}[/itex] [itex] g \frac{df}{dx} dx [/itex]

I'm choosing [itex]f = x\Psi^*[/itex] and [itex] g = \frac{\partial \Psi}{\partial x}[/itex], but I think I'm not getting right these limits considerations... any sugestions or enlightenments?_______________________________________________________
EDIT ([itex]\frac{\partial \Psi}{\partial}[/itex] with respect to time, not position)
Schrödinger Equation: [itex]i\hbar[/itex] [itex]\frac{\partial \Psi} {\partial t}[/itex] = -[itex] \frac{\hbar^2}{2m}[/itex] [itex]\frac{\partial^2 \Psi}{\partial x^2}[/itex] + [itex]V{} \Psi[/itex]
 
Last edited:
Physics news on Phys.org
  • #2
What happened to the dx/dt term when you took the time derivative of [itex]x\Psi\Psi^*[/itex]?
 
  • #3
vela said:
What happened to the dx/dt term when you took the time derivative of [itex]x\Psi\Psi^*[/itex]?

In QM x don't depend on t, right?
 
  • #4
*Fixed a typo in the equation
 
  • #5
emol1414 said:
In QM x don't depend on t, right?
You're right. The operator doesn't explicitly depend on time, so its derivative is 0.
 
  • #6
So, we have, for the product rule, that
[itex] \int_{a}^{b}[/itex] [itex] f \frac{dg}{dx} dx [/itex] = [itex] fg [/itex] [itex]{|}^{b}_{a}[/itex] - [itex] \int_{a}^{b}[/itex] [itex] g \frac{df}{dx} dx [/itex]

And choosing [itex]f = x\Psi^*[/itex] and [itex] g = \frac{\partial \Psi}{\partial x}[/itex]

[itex]\frac {d<x>}{dt}[/itex] = [itex]\frac{i \hbar}{2m}[/itex] {[itex]x \Psi^* \frac{\partial \Psi}{\partial x} |^{\infty}_{-\infty} - \int_{-\infty}^{\infty} \frac{\partial \Psi}{\partial x}(\Psi^* + x \frac{\partial \Psi^*}{\partial x})dx - x \Psi \frac{\partial \Psi^*}{\partial x} |^{\infty}_{-\infty} + \int_{-\infty}^{\infty} \frac{\partial \Psi^*}{\partial x}(\Psi + x \frac{\partial \Psi}{\partial x})dx[/itex]}

I guess up to this point it's ok... now I don't know how to work with these limits, which considerations should I do?
 
  • #7
Assume the wave function and its derivative go to 0 as x goes to ±∞.
 
  • #8
vela said:
Assume the wave function and its derivative go to 0 as x goes to ±∞.

Right... I'm not really sure why this is true (??), but doing so... we perform integration by parts 2 times and then

[itex]\frac {d<x>}{dt}[/itex] = -[itex]\frac{i \hbar}{2m}[/itex] {[itex] \int_{-\infty}^{\infty} \Psi^*(\frac{\partial \Psi}{\partial x}) - \Psi (\frac{\partial \Psi^*}{\partial x} )dx[/itex]}

That's it?
 
  • #9
You're almost done. Remember that[tex]\langle p \rangle = \int dx\,\Psi^* \hat{p} \Psi = \int dx\,\Psi^*\left(-i\hbar\frac{\partial}{\partial x}\right) \Psi[/tex]
You want to get the righthand side to look like that. One term already looks like that, but you still need to take care of the other one.
 
  • #10
Got it! Integration by parts only in one of the two terms left and then add to the other, so the factor 1/2 is gone... but... there's a m missing in the denominator, right?

Thank you! =)

One more thing... why is that [itex]\Psi[/itex] goes to 0 when x [itex]\rightarrow[/itex] [itex]\pm[/itex] [itex]\infty[/itex]? Is it a "single-case" fact, or is it always true?
 
  • #11
The wave function needs to vanish at infinity to be normalizable. You have to assume the function goes to 0 fast enough so that the boundary terms go to 0. There's probably a rigorous justification for it, but I don't recall it offhand.
 
  • #12
emol1414 said:
[...]

One more thing... why is that [itex]\Psi[/itex] goes to 0 when x [itex]\rightarrow[/itex] [itex]\pm[/itex] [itex]\infty[/itex]? Is it a "single-case" fact, or is it always true?

It's not mandatory, but usually one picks up from L^2 functions only the Schwartz test functions and that for a good reason.
 

Related to (Quantum Mechanics) Prove that <p> = m (d<x>/dt)

1. What is the equation

= m (d/dt) in quantum mechanics?

The equation

= m (d/dt) is known as the Heisenberg uncertainty principle in quantum mechanics. It relates the uncertainty in the momentum of a particle (

) to the uncertainty in its position (d/dt).

2. What is the significance of this equation in quantum mechanics?

The Heisenberg uncertainty principle is a fundamental principle in quantum mechanics that states that it is impossible to know the exact position and momentum of a particle simultaneously. This has important implications for the behavior and interactions of particles on a microscopic scale.

3. How is this equation derived in quantum mechanics?

The equation

= m (d/dt) can be derived from the commutation relation between the position and momentum operators in quantum mechanics. It is a consequence of the wave-like nature of particles on a quantum scale.

4. Can this equation be applied to all particles in quantum mechanics?

Yes, the Heisenberg uncertainty principle applies to all particles in quantum mechanics, including electrons, protons, and photons. It is a universal principle that governs the behavior of particles at a microscopic level.

5. How does this equation impact our understanding of the physical world?

The Heisenberg uncertainty principle challenges our traditional understanding of the physical world and introduces a new level of uncertainty into our observations and measurements. It has also led to the development of new theories and technologies, such as quantum computing, that utilize the principles of quantum mechanics.

Similar threads

  • Advanced Physics Homework Help
Replies
10
Views
619
  • Advanced Physics Homework Help
Replies
1
Views
1K
  • Advanced Physics Homework Help
Replies
30
Views
1K
  • Advanced Physics Homework Help
Replies
4
Views
968
  • Advanced Physics Homework Help
Replies
8
Views
3K
  • Advanced Physics Homework Help
Replies
5
Views
1K
  • Advanced Physics Homework Help
Replies
1
Views
1K
  • Advanced Physics Homework Help
Replies
1
Views
2K
  • Advanced Physics Homework Help
Replies
3
Views
1K
  • Advanced Physics Homework Help
Replies
3
Views
907
Back
Top